You are on page 1of 5

www.sakshieducation.

com

Geometrical Representation of Complex Numbers


Very Short Answer Questions
1. Show that the triangle formed with the points in the argand diagram
represented by ( 2 + 2i ) , − 2 − 2i − 2 3 + 2 3 i is equilateral triangle.

Solution: -

( )
Let A ( 2, 2 ) B ( −2, − 2 ) C −2 3, 2 3 be the points represented by the given
complex numbers 2 + 2i, − 2 − 2i − 2 3 + 2 3 i respectively

AB = ( 2 + 2) + ( 2 + 2 ) = 32
2 2

( −2 ) + (2 )
2 2
BC = 3+2 3+2 = 32

( −2 ) + (2 )
2 2
CA = 3−2 3−2 = 32

AB = BC = CA

∴ Triangle ABC is equilateral


2. Find the equation of the perpendicular bisector of the lines segment joining
the points 7 + 7i, 7 − 7i in the argand diagram

Solution: -
Let A(7, 7) B (7, - 7) be the ends of the line segment represented by the complex
numbers ( 7 + 7i ) and ( 7− 7i ) represents

Mid point of AB = ( 7, 0 )

−7 − 7
Slope of AB =
7−7

∴ Slope of perpendicular bisector = 0

Equal of perpendicular bisector is ( y − 0 ) = 0 ( x − 7 )

y=0

www.sakshieducation.com
www.sakshieducation.com

3. If z = x + iy and if the point P in the argand plane represents z1 find the


locus of z satisfying the equation

(i) | z − 2 − 3i | = 5 (ii) 2 | z− 2 | = | z − 1| (iii) Im ( z 2 ) = 4

Solution (i) | z − 2 − 3i | = 5

| z − 2 − 3i | = 5 ⇒ ( x − 2) + ( y − 3) = 5
2 2

x 2 + y 2 − 4 x − 6 y − 12 = 0

(ii) z | z − 2 | = | z − 1|

2 | ( x − 2 ) + iy | = | ( x− 1) + iy | ⇒ 2 ( x− 2 ) + y 2 = ( x − 1) + y 2
2 2

{ }
4 ( x −2 ) + y 2 = ( x − 1) + y 2 ⇒ 4 x 2 + 4 y 2 − 16 x + 16 = x 2 + y 2 − 2 x + 1
2 2

∴ 3x 2 + 3 y 2 −14 x + 15 = 0

(iii) ( )
Im z 2 = 4

z 2 = ( x + iy ) = x 2 − y 2 + 2ixy
2

( ) ( )
Im z 2 =2 xy {Im z 2 means Imaginary part of z 2 }

( )
im z 2 = 4 ⇒ 2 x y = 4 ⇒ xy = 2

SHORT ANSWER QUESTIONS


z3− z1
1. If is a real number then show that the points represented by the
z2 − z1
complex number z1 , z2 , z3 are collinear

Solution : -
z3− z1
Let = k where K is a real number
z2 − z1

z3− z1 = kz2 − kz1

kz1 − z1 = kz2 − z3

www.sakshieducation.com
www.sakshieducation.com

kz2 − z2
z1 =
k −1

i.e, z1 divides the line joining of z2 and z3

Externally in the ratio K :1

Hence z1 , z2 , z3 are collinear

2. Show that the four points in the argand plane represented by the complex
numbers z + i, 4 + 3i, 2+ 5i, 3i are the vertices of a square

Solution: -

Let A ( 2, 1) B ( 4, 3) C ( 2, 5 ) D ( 0,3) be the given vertices

AB = ( 4 − 2) + ( 3 − 1) = 8 BC = ( 2 −4 ) + ( 5 −3) = 8
2 2 2 2

CD = (0 − 2) + (3 − 5) = 8 AD = ( 0 −2 ) + ( 3 − 1) = 8
2 2 2 2

AC = ( 4 − 2) + ( 5 −1) = 4 BD = (0 − 4) + ( 3 − 3) = 4
2 2 2 2

Here AB = BC = CD = AD and AC = BD

Hence ABCD is a square


3. Show that the points represented by the complex number
−3 1 7
−2 + 7i, + i, 4 − 3i , (1 + i ) are the vertices of a rhombus
2 2 2
Solution ; -

 3 1 7 7
Let A ( −2, 7 ) B  − ,  C ( 4, − 3) D  ,  be the given vertices
 2 2 2 2
2 2 2 2
 3  1  170 7  7  170
AB =  − + 1 +  − 7  = CD =  − 4  +  + 3  =
 2  2  4 2  2  4

2 2
 3  1 170
BC =  4 +  +  −3 −  =
 2  2 4

2 2
7  7  170
AD =  + 2  +  − 7  =
2  2  4

www.sakshieducation.com
www.sakshieducation.com

AC = ( 4 + 2) + ( −3 − 7 ) = 136
2 2

AC = ( 4 + 2) + ( −3 − 7 ) = 136
2 2

AB = BC = CD = AD and AC ≠ BD

Hence ABCD is a rhombus


4. Show that the points in the argand diagram represented by the complex
numbers z1 , z2 , z3 are collinear if and only if there exists three real numbers
p, q, r not all zero satisfying pz1 + qz2 + rz3 = 0 and p + q + r = 0

Solution : -

Given pz1 + qz2 + r z3 = 0 and p +q + r = 0

∴ r = − p −q

∴ pz1 + qz2 + ( − p − q ) z3 = 0 {∵ r = − p − q}

pz1 + qz2 = ( p + q ) z3

qz2 + pz1
z3 =
q+ p

z3 divides the line joining of z1 and z2 in the ratio q : p

∴ z1 , z2 , z3 are collinear

Given z1 , z2 , z3 are collinear the

∴ Let z2 divide line joining of z1 & z3 in the ratio K : l

k z3 + l z1
∴ z2 =
k +l

( z + ( −k − l ) z
1 2 + k z3 = 0 )

This is of the form pz1 + qz2 + r z3 = 0

Where p = l q = − k − l ; r = k

p+q+r=0

Hence Proved

www.sakshieducation.com
www.sakshieducation.com

5. The points P, Q denote the complex numbers z1 z2 in the argand diagram. O


is the origin if z1 z2 + z1 z2 = 0 then show that POQ = 900

Solution : -

Let z1= x1 + iy1 and z2 = x2 + iy2

∴ P = ( x2 y1 ) Q = ( x2 y2 )

y1 y2
Slope of OP = slope of OQ =
x1 x2

Given z1 z2 + z1 z2 = 0

( x1 + iy1 )( x2 − iy2 ) + ( x1 − iy1 )( x2 + iy2 ) = 0


x1 x2 − i x1 y2 + i x2 y1 + y1 y2 + x1 x2 + i x1 y2 − i x2 y1 + y1 y2 = 0

x1 x2 + y1 y2 = 0 ⇒ = − x1 x2 = y1 y2

y1 y2
∴ × = −1
x1 x2

Slope of OP × slope of OQ = − 1

∴ POQ = 900

www.sakshieducation.com

You might also like